LSAT and Law School Admissions Forum

Get expert LSAT preparation and law school admissions advice from PowerScore Test Preparation.

 Administrator
PowerScore Staff
  • PowerScore Staff
  • Posts: 8917
  • Joined: Feb 02, 2011
|
#91407
Complete Question Explanation

The correct answer choice is (D).

Answer choice (A):

Answer choice (B):

Answer choice (C):

Answer choice (D): This is the correct answer choice.

Answer choice (E):

This explanation is still in progress. Please post any questions below!
User avatar
 christinecwt
  • Posts: 74
  • Joined: May 09, 2022
|
#95789
Hi Team - why Answer Choice B is wrong given that Answer Choice D is correct and it covers two of the days mentioned in Answer Choice B? Thanks!
User avatar
 katehos
PowerScore Staff
  • PowerScore Staff
  • Posts: 184
  • Joined: Mar 31, 2022
|
#95802
Hi christinecwt!

Answer choice (B) is incorrect because performing in Jazz on May 2, 4, and 5 creates a scenario that does not violate the rules, while placing Jazz on May 2 and 5 creates a scenario that does violate the rules. Since the question stem asks us to find out which schedule of Jazz Noguchi CANNOT perform in, the correct answer will be a placement of Jazz that violates the rules of the game.

Let's start with the scenario described in answer choice (B). If we place Jazz (J) on May 2, 4, and 5 only, we get a diagram that looks like this: _ J _ J J _. We can quickly place Orchestra (O) in spots 1 and 3 - which preserves the first rule - and place Solo (S) in spot 6. This gives us a valid scenario that ends up looking like: O J O J J S. So, we can eliminate (B)!

Moving on to answer choice (D), if we place J on May 2 and 5 only, then we get a diagram that looks like this: _ J _ _ J _. Now, if we try to place O in spots 1 and 3 - per the second rule - we get O J O _ J _. We can already see that this is an issue! We have an O without a J immediately following it, which violates the first rule. If we try putting O in spots 4 and 6, we run into the same issue (O cannot go in spot 6 since J must immediately follow O whenever O performs; the diagram would look like this before we cross it out _ J _ O J O). Since this is not a viable scenario, we know (D) is the correct answer, since Noguchi cannot perform Jazz on days 2 and 5 only!

I hope this helps :)
Kate
User avatar
 John1965
  • Posts: 2
  • Joined: Oct 12, 2022
|
#97838
Hi everyone,

I don’t understand Global question #11. Can anyone explain why A is incorrect?

Thanks,

John
 Adam Tyson
PowerScore Staff
  • PowerScore Staff
  • Posts: 5153
  • Joined: Apr 14, 2011
|
#97850
Sure thing, John! First, the question is asking for a negative - which of these answers CANNOT be true? That means the 4 wrong answers are all describing situations that COULD be true. They are all possible.

Next, notice that the answer choices all include the word "only"? Now imagine a scenario like answer A describes, where J is in slots 1, 3, and 4 only - there are no other Js in the solution. Could that work? Let's see:

We need at least one O somewhere, and it has to be right before a J. We can satisfy that by putting one O in slot 2. But we cannot add any more Os to the solution, because there is no other place to put an O right before a J.

Now if J is only in those three spaces, then spots 5 and 6 cannot be Js, so they would have to both have an S in them. That would give us this solution:

JOJJSS

Let's run that through the rules to see if it works:

1. Every O has a J right after it. Check!
2. Slots 1 and 3 are the same. Check!
3. There's never an S with an O right after it. Check!

Oh, and let's not forget the rule built into the scenario, that we have to use all three types. Check!

All of this proves that answer A COULD be true, and therefore it's a wrong answer because we want to select the one answer that CANNOT be true!

Get the most out of your LSAT Prep Plus subscription.

Analyze and track your performance with our Testing and Analytics Package.